K
Khách

Hãy nhập câu hỏi của bạn vào đây, nếu là tài khoản VIP, bạn sẽ được ưu tiên trả lời.

18 tháng 9 2017

 a) có nhiều cách chứng minh 
P = a/(b+c) + b/(c+a) + c/(a+b) 
P + 3 = 1+ a/(b+c) + 1+ b/(c+a) + 1+ c/(a+b) 
P + 3 = (a+b+c)/(b+c) + (a+b+c)/(b+c) + (a+b+c)/(c+a) 
P + 3 = (a+b+c)[1/(b+c) + 1/(c+a) + 1/(a+b)] (*) 

ad bđt cô si cho 3 số: 
2(a+b+c) = (a+b) + (b+c) + (c+a) ≥ 3.³√(a+b)(b+c)(c+a) 
1/(b+c) + 1/(c+a) + 1/(a+b) ≥ 3.³√1/(a+b)(b+c)(c+a) 

nhân lại vế theo vế 2 bđt: 2(a+b+c)[1/(b+c) + 1/(c+a) + 1/(a+b)] ≥ 9 
=> P + 3 ≥ 9/2 => P ≥ 3/2 (đpcm) ; dấu "=" khi a = b = c 
- - - 
cách khác: P = a/(b+c) + b/(c+a) + c/(a+b) 
M = b/(b+c) + c/(c+a) + a/(a+b) 
N = c/(b+c) + a/(c+a) + b/(a+b) 

Thấy: M + N = 3 
P + M = (a+b)/(b+c) + (b+c)/(c+a) + (c+a)/(a+b) ≥ 3 (cô si cho 3 số) 
P + N = (a+c)/(b+c) + (b+a)/(c+a) + (c+b)/(a+b) ≥ 3 (cô si) 

=> 2P + M + N ≥ 6 => 2P + 3 ≥ 6 => P ≥ 3/2 (đpcm) ; đẳng thức khi a = b = c 
-------------- 
b) ad bđt Bunhia: 1² = [2.(2x) + 1.y]² ≤ (2²+1²)(4x²+y²) => 4x² + y² ≥ 1/5 (đpcm) 
dấu "=" khi 2x/2 = y/1 và 4x+y = 1 <=> x = y = 1/5 
- - - 
Có thể không cần Bunhia, ad bđt a² + b² ≥ 2ab (*) 
(*) quá hiển nhiên từ (a-b)² ≥ 0 
x² + 1/25 ≥ 2x/5 <=> 4x² ≥ 8x/5 - 4/25 (1*) 
y² + 1/25 ≥ 2y/5 <=> y² ≥ 2y/5 - 1/25 (2*) 

lấy (1*)+(2*) => 4x²+y² ≥ 8x/5+2y/5 - 4/25 - 1/25 = 2(4x+y)/5 - 5/25 = 1/5 (đpcm) 
dấu "=" khi x = y = 1/5 
------------- 
c) ad bđt cô si cho 3 số: 
ab/c + bc/a + ca/b ≥ 3.³√(ab/c)(bc/a)(ca/b) = 3.³√abc 
- - - - 
nếu như đề đã ghi, thay a = b = c = 2 thì VT = 2+2+2 = 6 < VP = 2.2.2 = 8  

19 tháng 9 2017

*C/m: \(9(a+b)(b+c)(c+a)\ge8(a+b+c)(ab+bc+ac)\)

\(\Leftrightarrow a^2b+ab^2+a^2c+ac^2+b^2c+bc^2-6abc\ge0\)

\(VT=a^2b+ab^2+a^2c+ac^2+b^2c+bc^2\ge6\sqrt[6]{\left(abc\right)^6}=VP\)

Khi \(a=b=c\)

# Dôn lùng đợi chiều tối về t giải phần căn bậc 4 kia cho :)

8 tháng 1 2020

\(\left(a+b\right)\left(b+c\right)\left(c+a\right)+abc\)

\(=abc+a^2b+ab^2+a^2c+ac^2+b^2c+bc^2+abc+abc\)

\(=\left(a+b+c\right)\left(ab+bc+ca\right)\)( phân tích nhân tử các kiểu )

\(\Rightarrow\left(a+b\right)\left(b+c\right)\left(c+a\right)\ge\left(a+b+c\right)\left(ab+bc+ca\right)-abc\left(1\right)\)

\(a+b+c\ge3\sqrt[3]{abc};ab+bc+ca\ge3\sqrt[3]{a^2b^2c^2}\Rightarrow\left(a+b+c\right)\left(ab+bc+ca\right)\ge9abc\)

\(\Rightarrow-abc\ge\frac{-\left(a+b+c\right)\left(ab+bc+ca\right)}{9}\)

Khi đó:\(\left(a+b+c\right)\left(ab+bc+ca\right)-abc\)

\(\ge\left(a+b+c\right)\left(ab+bc+ca\right)-\frac{\left(a+b+c\right)\left(ab+bc+ca\right)}{9}\)

\(=\frac{8\left(a+b+c\right)\left(ab+bc+ca\right)}{9}\left(2\right)\)

Từ ( 1 ) và ( 2 ) có đpcm

11 tháng 11 2019

2/ Không mất tính tổng quát, giả sử \(c=min\left\{a,b,c\right\}\).

Nếu abc = 0 thì có ít nhất một số bằng 0. Giả sử c = 0. BĐT quy về: \(a^2+b^2\ge2ab\Leftrightarrow\left(a-b\right)^2\ge0\) (luôn đúng)

Đẳng thức xảy ra khi a = b; c = 0.

Nếu \(abc\ne0\). Chia hai vế của BĐT cho \(\sqrt[3]{\left(abc\right)^2}\)

BĐT quy về: \(\Sigma_{cyc}\sqrt[3]{\frac{a^4}{b^2c^2}}+3\ge2\Sigma_{cyc}\sqrt[3]{\frac{ab}{c^2}}\)

Đặt \(\sqrt[3]{\frac{a^2}{bc}}=x;\sqrt[3]{\frac{b^2}{ca}}=y;\sqrt[3]{\frac{c^2}{ab}}=z\Rightarrow xyz=1\)

Cần chúng minh: \(x^2+y^2+z^2+3\ge2\left(\frac{1}{x}+\frac{1}{y}+\frac{1}{z}\right)\)

\(\Leftrightarrow x^2+y^2+z^2+2xyz+1\ge2\left(xy+yz+zx\right)\) (1)

Theo nguyên lí Dirichlet thì trong 3 số x - 1, y - 1, z - 1 tồn tại ít nhất 2 số có tích không âm. Không mất tính tổng quát, giả sử \(\left(x-1\right)\left(y-1\right)\ge0\)

\(\Rightarrow2xyz\ge2xz+2yz-2z\). Thay vào (1):

\(VT\ge x^2+y^2+z^2+2xz+2yz-2z+1\)

\(=\left(x-y\right)^2+\left(z-1\right)^2+2xy+2xz+2yz\)

\(\ge2\left(xy+yz+zx\right)\)

Vậy (1) đúng. BĐT đã được chứng minh.

Đẳng thức xảy ra khi a = b = c hoặc a = b, c = 0 và các hoán vị.

Check giúp em vs @Nguyễn Việt Lâm, bài dài quá:(

6 tháng 7 2020

Để đưa về chứng minh $(1)$ và $(2)$ ta dùng:

Định lí SOS: Nếu \(X+Y+Z=0\) thì \(AX^2+BY^2+CZ^2\ge0\)

khi \(\left\{{}\begin{matrix}A+B+C\ge0\\AB+BC+CA\ge0\end{matrix}\right.\)

Chứng minh: Vì \(\sum\left(A+C\right)=2\left(A+B+C\right)\ge0\)

Nên ta có thể giả sử \(A+C\ge0\). Mà $X+Y+Z=0$ nên$:$

\(AX^2+BY^2+CZ^2=AX^2+BY^2+C\left[-\left(X+Y\right)\right]^2\)

\(={\frac { \left( AX+CX+CY \right) ^{2}}{A+C}}+{\frac {{Y}^{2} \left( AB+AC+BC \right) }{A+C}} \geq 0\)

4 tháng 2 2021

jjjjjjjjjjjjjjjjjjjjjjjjjjjjjjjjjjjjjjjjjjjjjjjjjjjjjjjjjjjjjjjjjjjjjjjjjjjjjjjjjjjjjjjjjjjjjjjjjjjjjjjjjjjjjjjjjjjjjjjjjjjjjjjjjjjjjjjjjjjjjjjjjjjjjjjjjjjjjjjjjjjjjjjjjjjjjjjjjjjjjjjjjjjjjjjjjjjjjjjjjjjjjjjjjjjjjjjjjjjjjjjjjjjjjjjjjjjjjjjjjjjjjjjjjjjjjjjjjjjjjjjjjjjjjjjjjjjjjjjjjjjjjjjjjjjjjjjjjjjjjjjjjjjjjjjjjjjjjjjjjjjjjjjjjjjjjjjjjjjj

4 tháng 2 2021

OMG !!!!!!!!!!!!!!!!!!!!!!!!!!!!!!!!!!!!!!!

6 tháng 10 2019

\(VT=\frac{ab+bc+ca}{ab}+\frac{ab+bc+ca}{bc}+\frac{ab+bc+ca}{ca}\)

\(=3+\frac{c\left(a+b\right)}{ab}+\frac{a\left(b+c\right)}{bc}+\frac{b\left(c+a\right)}{ca}\)(1)

Theo BĐT AM-GM: \(\frac{1}{2}\left[\frac{c\left(a+b\right)}{ab}+\frac{a\left(b+c\right)}{bc}\right]\ge\sqrt{\frac{\left(a+b\right)\left(b+c\right)}{b^2}}\)

Tương tự: \(\frac{1}{2}\left[\frac{a\left(b+c\right)}{bc}+\frac{b\left(c+a\right)}{ca}\right]\ge\sqrt{\frac{\left(a+c\right)\left(b+c\right)}{c^2}}\)

\(\frac{1}{2}\left[\frac{c\left(a+b\right)}{ab}+\frac{b\left(c+a\right)}{ca}\right]\ge\sqrt{\frac{\left(a+c\right)\left(a+b\right)}{a^2}}\)

Cộng theo vế 3 BĐT trên rồi thay vào 1 ta sẽ thu được đpcm.

6 tháng 10 2019

Ý em là thay vào (1) !!

NV
11 tháng 2 2020

Mới nghĩ ra 3 câu:

a/ \(\frac{ab}{\sqrt{\left(1-c\right)^2\left(1+c\right)}}=\frac{ab}{\sqrt{\left(a+b\right)^2\left(1+c\right)}}\le\frac{ab}{2\sqrt{ab\left(1+c\right)}}=\frac{1}{2}\sqrt{\frac{ab}{1+c}}\)

\(\sum\sqrt{\frac{ab}{1+c}}\le\sqrt{2\sum\frac{ab}{1+c}}\)

\(\sum\frac{ab}{1+c}=\sum\frac{ab}{a+c+b+c}\le\frac{1}{4}\sum\left(\frac{ab}{a+c}+\frac{ab}{b+c}\right)=\frac{1}{4}\)

c/ \(ab+bc+ca=2abc\Rightarrow\frac{1}{a}+\frac{1}{b}+\frac{1}{c}=2\)

Đặt \(\left(x;y;z\right)=\left(\frac{1}{a};\frac{1}{b};\frac{1}{c}\right)\Rightarrow x+y+z=2\)

\(VT=\sum\frac{x^3}{\left(2-x\right)^2}\)

Ta có đánh giá: \(\frac{x^3}{\left(2-x\right)^2}\ge x-\frac{1}{2}\) \(\forall x\in\left(0;2\right)\)

\(\Leftrightarrow2x^3\ge\left(2x-1\right)\left(x^2-4x+4\right)\)

\(\Leftrightarrow9x^2-12x+4\ge0\Leftrightarrow\left(3x-2\right)^2\ge0\)

d/ Ta có đánh giá: \(\frac{x^4+y^4}{x^3+y^3}\ge\frac{x+y}{2}\)

\(\Leftrightarrow\left(x-y\right)^2\left(x^2+xy+y^2\right)\ge0\)

11 tháng 2 2020

Akai Haruma, Nguyễn Ngọc Lộc , @tth_new, @Băng Băng 2k6, @Trần Thanh Phương, @Nguyễn Việt Lâm

Mn giúp e vs ạ! Thanks!

30 tháng 11 2019

Nguyễn Việt Lâm anh làm bài này giúp em với ạ

30 tháng 11 2019

Akai Haruma giúp em bài trên với ạ

1. a) \(\left\{{}\begin{matrix}x,y,z0\\xyz=1\end{matrix}\right.\). Tìm max \(P=\frac{1}{\sqrt{x^5-x^2+3xy+6}}+\frac{1}{\sqrt{y^5-y^2+3yz+6}}+\frac{1}{\sqrt{z^5-z^2+zx+6}}\) b) \(\left\{{}\begin{matrix}x,y,z0\\xyz=8\end{matrix}\right.\). Min \(P=\frac{x^2}{\sqrt{\left(1+x^3\right)\left(1+y^3\right)}}+\frac{y^2}{\sqrt{\left(1+y^3\right)\left(1+z^3\right)}}+\frac{z^2}{\sqrt{\left(1+z^3\right)\left(1+x^3\right)}}\) c) \(x,y,z0.\) Min...
Đọc tiếp

1. a) \(\left\{{}\begin{matrix}x,y,z>0\\xyz=1\end{matrix}\right.\). Tìm max \(P=\frac{1}{\sqrt{x^5-x^2+3xy+6}}+\frac{1}{\sqrt{y^5-y^2+3yz+6}}+\frac{1}{\sqrt{z^5-z^2+zx+6}}\)

b) \(\left\{{}\begin{matrix}x,y,z>0\\xyz=8\end{matrix}\right.\). Min \(P=\frac{x^2}{\sqrt{\left(1+x^3\right)\left(1+y^3\right)}}+\frac{y^2}{\sqrt{\left(1+y^3\right)\left(1+z^3\right)}}+\frac{z^2}{\sqrt{\left(1+z^3\right)\left(1+x^3\right)}}\)

c) \(x,y,z>0.\) Min \(P=\sqrt{\frac{x^3}{x^3+\left(y+z\right)^3}}+\sqrt{\frac{y^3}{y^3+\left(z+x\right)^3}}+\sqrt{\frac{z^3}{z^3+\left(x+y\right)^3}}\)

d) \(a,b,c>0;a^2+b^2+c^2+abc=4.Cmr:2a+b+c\le\frac{9}{2}\)

e) \(\left\{{}\begin{matrix}a,b,c>0\\a+b+c=3\end{matrix}\right.\). Cmr: \(\frac{a}{b^3+ab}+\frac{b}{c^3+bc}+\frac{c}{a^3+ca}\ge\frac{3}{2}\)

f) \(\left\{{}\begin{matrix}a,b,c>0\\ab+bc+ca+abc=4\end{matrix}\right.\) Cmr: \(\sqrt{ab}+\sqrt{bc}+\sqrt{ca}\le3\)

g) \(\left\{{}\begin{matrix}a,b,c>0\\ab+bc+ca+abc=2\end{matrix}\right.\) Max : \(Q=\frac{a+1}{a^2+2a+2}+\frac{b+1}{b^2+2b+2}+\frac{c+1}{c^2+2c+2}\)

3
26 tháng 4 2020

Câu 1 chuyên phan bội châu

câu c hà nội

câu g khoa học tự nhiên

câu b am-gm dựa vào hằng đẳng thử rồi đặt ẩn phụ

câu f đặt \(a=\frac{2m}{n+p};b=\frac{2n}{p+m};c=\frac{2p}{m+n}\)

Gà như mình mấy câu còn lại ko bt nha ! để bạn tth_pro full cho nhé !

25 tháng 4 2020

Câu c quen thuộc, chém trước:

Ta có BĐT phụ: \(\frac{x^3}{x^3+\left(y+z\right)^3}\ge\frac{x^4}{\left(x^2+y^2+z^2\right)^2}\) \((\ast)\)

Hay là: \(\frac{1}{x^3+\left(y+z\right)^3}\ge\frac{x}{\left(x^2+y^2+z^2\right)^2}\)

Có: \(8(y^2+z^2) \Big[(x^2 +y^2 +z^2)^2 -x\left\{x^3 +(y+z)^3 \right\}\Big]\)

\(= \left( 4\,x{y}^{2}+4\,x{z}^{2}-{y}^{3}-3\,{y}^{2}z-3\,y{z}^{2}-{z}^{3 } \right) ^{2}+ \left( 7\,{y}^{4}+8\,{y}^{3}z+18\,{y}^{2}{z}^{2}+8\,{z }^{3}y+7\,{z}^{4} \right) \left( y-z \right) ^{2} \)

Từ đó BĐT \((\ast)\) là đúng. Do đó: \(\sqrt{\frac{x^3}{x^3+\left(y+z\right)^3}}\ge\frac{x^2}{x^2+y^2+z^2}\)

\(\therefore VT=\sum\sqrt{\frac{x^3}{x^3+\left(y+z\right)^3}}\ge\sum\frac{x^2}{x^2+y^2+z^2}=1\)

Done.